Old Q&A – Nephrology

 

A 7-year-old boy is brought to the emergency department by his mother because of “tea-colored urine” for the last several days. He has also had some nausea and vomiting, and his eyes appear swollen when he wakes up in the morning. The eye swelling tends to resolve over the course of the day. He is generally very healthy and there is no family history of any chronic diseases. His temperature is 36.7 C (98.0 F), blood pressure is 130/90 mm Hg, pulse is 96/min, and respiratory rate is 16/min. Physical examination is unremarkable. A urinalysis shows red cell casts. At this time the most appropriate study to confirm your diagnosis is

  A. antinuclear antibody
  B. antistreptolysin O antibody
  C. renal biopsy
  D. renal ultrasound
  E. urine culture
Explanation:

The correct answer is B. Poststreptococcal glomerulonephritis is the most common cause of acute glomerulonephritis in children. It usually follows a streptococcal pharyngitis by 1-2 weeks and a streptococcal skin infection by 2-3 weeks. It most commonly occurs in school-age children and has a male to female predominance of 2:1. It is most commonly characterized by hematuria (microscopic or gross) with red cell casts, proteinuria, hypertension (from fluid overload secondary to decreased glomerular filtration rate), and edema (from retention of salt and water). Laboratory values are usually significant for markedly decreased complement levels (C3 and C4), hypo- or hypernatremia, and a blood urea nitrogen elevated disproportionately to the creatinine. In order to diagnose poststreptococcal glomerulonephritis with certainty, there needs to be evidence of a preceding streptococcal infection such as an elevated ASO or streptozyme.

Systemic lupus erythematosus (SLE) nephritis is another cause of hematuria. If this child had persistently low complement levels, an ANA (choice A) would be indicated as one of the first screening tests for SLE. Keep in mind that only 25% of cases of SLE are diagnosed within the first two decades of life, the male-to-female ratio is 1:8, and renal disease is not present in all cases of SLE. Therefore, it is unlikely that this 7-year-old boy has new onset SLE, and other more common causes of glomerulonephritis must be ruled out first.

At this time, a renal biopsy (choice C) is not indicated. A renal biopsy is an invasive procedure with inherent risks. As this patient fits the perfect description for post streptococcal glomerulonephritis a renal biopsy would only be indicated if the complement level does not return to normal within 8 weeks, and the gross hematuria does not resolve over several weeks. If one was to do a renal biopsy on a patient with post-streptococcal glomerulonephritis, the findings would be: mesangial and capillary cell proliferation, inflammatory cell infiltration, and granular “humps” of IgG and C3 below the glomerular basement membrane.

A renal ultrasound (choice D) is useful in evaluating parenchymal disease, urinary tract abnormalities, or renal blood flow. In the case of post-streptococcal glomerulonephritis, the changes are microscopic and therefore would not be seen on ultrasound.

Urinary tract infections are a common cause of hematuria. Therefore, it is a good idea to do a urine culture (choice E) in all cases of hematuria. Unfortunately, the child in this case has red cell casts which indicate the kidneys are the source for the blood. So, although a urine culture should be sent on this child, it will probably not help with the diagnosis.

 

A 34-year-old woman comes to the office for a follow-up examination after passing a kidney stone in the hospital last week. You were away on vacation and so your partner was involved in her in-patient treatment. The patient tells you that your partner did not tell her anything about her condition and always seemed “as if he was late for his golf tee-off time.” Passing the stone was “more painful than the vaginal delivery of all 3 children combined” and so she wants to make sure that she never has one again. She has no chronic medical conditions, never had surgery, and takes no medications. Her father and brother both suffer from nephrolithiasis. You go over to the computer to check if the laboratory report on the composition of her kidney stone is complete. You see that the stone was composed of calcium and that she had a 24-hour urine collection done in the hospital that showed 295 mg of calcium and 15 mg of oxalate. Her serum calcium level is 8.5 mg/dL. The most appropriate course of action is to

  A. prescribe allopurinol, orally
  B. prescribe cholestyramine, orally, and a low-fat diet
  C. prescribe hydrochlorothiazide, orally
  D. recommend a high fluid intake, but no medications or supplements
  E. recommend megadoses of vitamin C
Explanation:

The correct answer is C. This patient most likely has idiopathic hypercalcuria, which is characterized by calcium kidney stones, normocalcemia, and unexplained hypercalcuria. It is believed to be hereditary and is more common in men. Hydrochlorothiazide has been shown to decrease urinary calcium levels and therefore decrease the rate of calcium stone formation.

Allopurinol (choice A) is a xanthine oxidase inhibitor that is used in patients with kidney stones composed of uric acid. It decreases serum and urinary uric acid levels.

Cholestyramine (choice B), an oxalate binding resin, combined with a low-fat diet, is used in patients with intestinal hyperoxaluria for the prevention of calcium stones. These patients have a 24-hour urinary oxalate level greater than 50 mg. It occurs in patients with prior bowel surgery and chronic diseases of the gastrointestinal tract that lead to malabsorption and bacterial overgrowth.

It is inappropriate to recommend a high fluid intake, with no medications or supplements (choice D) to a patient with idiopathic hypercalcuria and kidney stones. Thiazides have been shown to reduce the rate of kidney stone formation in a patient with her condition.

Megadoses of vitamin C (choice E) are controversial and have been associated with the formation of kidney stones. Megadoses of vitamin C are not generally recommended for the prevention of calcium stones in patients with idiopathic hypercalcuria.

 

  A 38-year-old man is admitted to the hospital for acute deterioration in renal function. He was seen in your office 2 days prior for some mild upper respiratory complaints, including a sore throat, cough, and fever. He was prescribed cephalexin and sent home. Today, his laboratory data returned and shows a blood urea nitrogen level of 67 mg/dL and a creatinine level of 2.1 mg/dL. You called him and told him to meet you at the hospital for further evaluation. On admission his BUN is now 109 mg/dL and his creatinine is 4.2 mg/dL. The appropriate tests are ordered and an electrocardiogram shows QRS complex widening and tall, peaked T waves. His temperature is 38.3 C (101.0 F). He has an erythematous oropharynx with some mild tonsillar exudate. His lungs are clear. It is observed that he has urinated only 5-10 cc in the past 2 hours since his hospitalization. A urinalysis shows red cell casts and dysmorphic red blood cells. The most appropriate next step is to

  A. administer high-dose methyl-prednisolone, intravenously
  B. administer low-dose methyl-prednisolone, intravenously
  C. administer penicillin, intravenously
  D. prescribe high-dose cyclophosphamide, orally
  E. prescribe penicillin, orally
Explanation:

The correct answer is A. The diagnosis and treatments for rapidly progressive glomerulonephritis, in this case, are woefully inadequate. In adults, where the disease is much less common, and the outcome much worse, early initiation of high-dose steroids has been associated with improved mortality and more rapid return of renal function. Even this therapy, however, has benefits for at most, 40% of patients.

For this reason, lower dose steroids (choice B) are not adequate therapy.

Intravenous penicillin (choice C) or oral penicillin (choice E) presume that treatment of the inciting infection will alter the course of the renal failure. This has not been the case in clinical practice and the severity of the precipitating illness and its duration are not correlated with the course of the glomerulonephritis.

Oral high-dose cyclophosphamide (choice D) is often reserved for more dramatic immune suppression in patients with more severe disease, but its efficacy and applicability to all RPGN patients is far from clear and therefore the risks associated with its use are not offset by any benefit.

 

A 7-year-old boy is brought to the emergency department by his mother because of “tea-colored urine” for the last several days. He has also had some nausea and vomiting, and his eyes appear swollen when he wakes up in the morning. The eye swelling tends to resolve over the course of the day. He is generally very healthy and there is no family history of any chronic diseases. His temperature is 36.7 C (98.0 F), blood pressure is 130/90 mm Hg, pulse is 96/min, and respiratory rate is 16/min. Physical examination is unremarkable. A urinalysis shows red cell casts. At this time the most appropriate study to confirm your diagnosis is

  A. antinuclear antibody
  B. antistreptolysin O antibody
  C. renal biopsy
  D. renal ultrasound
  E. urine culture
Explanation:

The correct answer is B. Poststreptococcal glomerulonephritis is the most common cause of acute glomerulonephritis in children. It usually follows a streptococcal pharyngitis by 1-2 weeks and a streptococcal skin infection by 2-3 weeks. It most commonly occurs in school-age children and has a male to female predominance of 2:1. It is most commonly characterized by hematuria (microscopic or gross) with red cell casts, proteinuria, hypertension (from fluid overload secondary to decreased glomerular filtration rate), and edema (from retention of salt and water). Laboratory values are usually significant for markedly decreased complement levels (C3 and C4), hypo- or hypernatremia, and a blood urea nitrogen elevated disproportionately to the creatinine. In order to diagnose poststreptococcal glomerulonephritis with certainty, there needs to be evidence of a preceding streptococcal infection such as an elevated ASO or streptozyme.

Systemic lupus erythematosus (SLE) nephritis is another cause of hematuria. If this child had persistently low complement levels, an ANA (choice A) would be indicated as one of the first screening tests for SLE. Keep in mind that only 25% of cases of SLE are diagnosed within the first two decades of life, the male-to-female ratio is 1:8, and renal disease is not present in all cases of SLE. Therefore, it is unlikely that this 7-year-old boy has new onset SLE, and other more common causes of glomerulonephritis must be ruled out first.

At this time, a renal biopsy (choice C) is not indicated. A renal biopsy is an invasive procedure with inherent risks. As this patient fits the perfect description for post streptococcal glomerulonephritis a renal biopsy would only be indicated if the complement level does not return to normal within 8 weeks, and the gross hematuria does not resolve over several weeks. If one was to do a renal biopsy on a patient with post-streptococcal glomerulonephritis, the findings would be: mesangial and capillary cell proliferation, inflammatory cell infiltration, and granular “humps” of IgG and C3 below the glomerular basement membrane.

A renal ultrasound (choice D) is useful in evaluating parenchymal disease, urinary tract abnormalities, or renal blood flow. In the case of post-streptococcal glomerulonephritis, the changes are microscopic and therefore would not be seen on ultrasound.

Urinary tract infections are a common cause of hematuria. Therefore, it is a good idea to do a urine culture (choice E) in all cases of hematuria. Unfortunately, the child in this case has red cell casts which indicate the kidneys are the source for the blood. So, although a urine culture should be sent on this child, it will probably not help with the diagnosis.

 

A 56-year-old man comes to the emergency department with severe right flank pain for the past 5 days. He says that the pain started after he returned from a long hiking trip in the Grand Canyon, and despite taking some ibuprofen, it has not improved. Infact, he came to the hospital today because the pain has increased and he now has new onset of nausea and chills. He tells you that he had similar pain several years ago that was diagnosed as uric acid stones. He was treated conservatively and eventually passed all the stones spontaneously. His temperature is 38.1 C (100.6 F), blood pressure is 130/80 mm Hg, and pulse is 115/min. On examination, the patient is unable to lie still because of the pain and has significant right costovertebral tenderness radiating to his right testicle. Leukocyte count is 16,000/mm3 and his creatinine is 2.1 mg/dL. The most appropriate study after starting the patient on intravenous hydration and antibiotics is

  A. a CT scan of the abdomen and pelvis
  B. a DMSA renal scan
  C. an intravenous pyelogram
  D. a radiograph of the kidneys, ureters, and bladder
  E. a renal ultrasound
Explanation:

The correct answer is A. The patient’s symptoms are consistent with an infected and obstructing ureteral stone, especially in light of his past stone history and probable dehydration from his hiking trip. Colicky flank pain radiating to the groin typically marks the presence of a midureteral obstruction. An obstruction in the presence of an infection is a medical emergency because of the very high possibility of septic shock, therefore a quick diagnosis is needed. The best imaging study for nephrolithiasis is a non-contrast CT scan which can visualize all types of stones and then accurately locate them. It can also tell us whether the patient has hydronephrosis, hence, disclosing whether the patient has an obstruction. Once the diagnosis is made, a urologist will most likely need to put in a ureteral stent to correct the obstruction.

A DMSA renal scan (choice B) is a radionucleotide study that evaluates renal function and does not give much information about a renal obstruction. It also takes 4-24 hours to obtain optimal images.

An IVP (choice C) is incorrect because of the patient’s renal insufficiency at the time of presentation. The elevated creatinine is a contraindication to IV contrast. In any other cases of noninfected nephrolithiasis with a normal creatinine, an IVP would be appropriate.

A KUB (choice D) would not be able to demonstrate the presence of a renal obstruction. It can be helpful in verifying the presence of kidney stones. However, uric acid stones are radiolucent and would not be seen on a plain film.

A renal ultrasound (choice E) is incorrect because an ultrasound cannot visualize the presence of ureteral stones. It can tell us whether the patient has hydronephrosis, but nothing more.

 

A 63-year-old postmenopausal woman comes to the clinic for a routine periodic health maintenance examination. She is slightly overweight with hypertension and type II diabetes, both of which are well controlled on medication. She also has a history of recurrent urinary tract infections and has been treated several times in the past with antibiotics which take care of her symptoms of dysuria and frequency. She is currently asymptomatic and has not had an infection in the last few months. Routine blood tests, including cholesterol levels, are all normal. A urinalysis shows:

A urine culture is then sent, which returns negative after 2 days. The most appropriate next step is to

  A. check another urinalysis today
  B. order a CT urogram and refer her to a urologist
  C. order a renal ultrasound and refer her to a urologist
  D. prescribe a 7-day course of ciprofloxacin and repeat the urinalysis after treatment
  E. reexamine her in 1 week
Explanation:

The correct answer is B. The patient has microscopic hematuria without evidence of a urinary tract infection. The correct work up for a painless hematuria, whether it is microscopic or gross, is a contrast study of the upper tract (CT urogram or an IVP) to look at the kidneys and ureters and a cystoscopy to look in the bladder to rule out any neoplasms.

Checking another urinalysis today (choice A) is incorrect because another urinalysis would not change the management of this patient. Even if the next urinalysis does not reveal any RBCs, the presence of blood in this current urinalysis dictates that an appropriate work up be performed. The patient may also have intermittent hematuria, which may be missed on subsequent urinalysis.

An ultrasound and referral (choice C) is incorrect. Though, a renal ultrasound can pick up abnormalities in the kidney including hydronephrosis or a tumor, it cannot be used to evaluate the ureters.

Antibiotics and repeat urinalysis (choice D) is incorrect, because the patient does not have any evidence of an infection.

Reexamining her in 1-week (choice E) is incorrect because painless hematuria needs to be worked up to rule out urological malignancies.

A 29-year-old woman, who has been undergoing treatment for hypertension for the past 2 years, comes to the office because of chills and right-sided flank pain. She has had chronic “low back pain” that you have been treating unsuccessfully with nonsteroidal anti-inflammatory drugs. She has never had any diagnostic studies performed to evaluate her hypertension or back pain. She is estranged from her family, but she knows that her mother and brother have been treated for hypertension starting at age 25. She has not spoken to them in 10 years and so she is unaware of any other medical conditions. Her temperature is 38.1 C (100.6 F), blood pressure is 130/90 mm Hg, and pulse is 65/min. On physical examination today there is marked right-sided flank tenderness. Cardiac examination reveals a mid-systolic click. Urinalysis shows pyuria and white blood cell casts. You prescribe a 14-day course of trimethoprim-sulfamethoxazole, schedule a renal ultrasound and a follow-up visit. You tell her to call your office immediately if the symptoms worsen. On the return visit, she says that she feels much better, but still has dull flank pain. Her temperature is 37.0 C (98.6 F). The ultrasound report is in the chart and states that there are 7 cysts in her right kidney and 5 cysts in her left kidney. At this time the most correct statement about her condition is:

  A. Her hypertension is unrelated to the kidney disease
  B. Long-term treatment with trimethoprim-sulfamethoxazole will eradicate the cysts
  C. She has a 20% chance of developing end-stage renal disease by age 70
  D. She is at an increased risk for developing colonic diverticular disease with perforation
  E. There is a 70% chance that she has an intracranial aneurysm and will suffer from a subarachnoid hemorrhage
Explanation:

The correct answer is D. This patient most likely has polycystic kidney disease and one of the most common extrarenal manifestations is colonic diverticular disease with an increased risk of perforation. The exact cause of this increased risk of perforation is not known. The hypertension, chronic flank pain, and the ultrasound findings are diagnostic for polycystic kidney disease. These patients tend to develop urinary tract infections, including acute pyelonephritis.

Her hypertension is related to the polycystic kidney disease and therefore it is incorrect to say that it is unrelated to the kidney disease (choice A). It is most likely caused by the activation of the renin-angiotensin system secondary to the ischemia caused by the distortion of the renal architecture by the cysts.

It is incorrect to say that long-term treatment with trimethoprim-sulfamethoxazole will eradicate the cysts (choice B) because the cysts are most likely caused by polycystic kidney disease, which is an autosomal dominant disease that cannot be treated with an antibiotics. Antibiotics may be used to treat infections, but not to eradicate the cysts.

This patient has approximately a 50% chance of developing end stage renal disease by age 70, not a 20% chance (choice C). Polycystic kidney disease is associated with slowly progressive renal failure. The factors that are associated with an earlier onset of renal failure are male sex, hypertension, early age at diagnosis, and multiple infections.

Only about 10-15% of patients with polycystic kidney disease have an intracranial aneurysm and will suffer from a subarachnoid hemorrhage, not 70% (choice E). Many exam questions ask about subarachnoid hemorrhages in patients with polycystic kidney disease so it seems more prevalent, but they are actually not very common.

 

A 1-day-old boy is in the intensive care nursery with renal failure and respiratory distress. On prenatal ultrasound, he had hydroureteronephrosis and a large bladder. Late in the prenatal course, oligohydramnios developed prompting delivery. During his first day of life, there had been no urine output and there was difficulty placing a bladder catheter. On physical examination, there is a palpable mass in the lower abdomen, presumed to be the bladder seen on prior ultrasound examination. Laboratory studies reveal a creatinine of 2.4 mg/dL and a blood urea nitrogen of 42 mg/dL. The most appropriate diagnostic study to order at this time is a (n)

  A. CT scan of the abdomen and pelvis
  B. Gastrografin enema study
  C. MRI of the abdomen and pelvis
  D. renal ultrasound examination
  E. voiding cystourethrogram
Explanation:

The correct answer is E. This is a classic presentation for posterior urethral valves in a newborn. The syndrome of posterior urethral valves basically is caused by varying degrees of obstruction in the posterior urethra in males by cusps of tissue called valves. This basically causes dilation of the bladder due to increased pressures and this pressure is transmitted up into the collecting systems of the kidneys, and can result in varying degrees of renal damage. The way to make the diagnosis is with a voiding cystourethrogram that will show a thickened and trabeculated bladder from muscular hypertrophy, (since the bladder has been squeezing against a tight obstruction), and a dilated posterior urethra with a narrowing. Treatment is surgical; a procedure called fulguration of the valves.

A CT scan (choice A) or MRI (choice C) of the abdomen and pelvis as well as a renal ultrasound (choice D) would tell you what you already know from the prenatal ultrasound, and that is that he has an enlarged, trabeculated bladder along with bilateral hydroureteronephrosis. None of these studies will adequately image the urethra.

A Gastrografin enema study (choice B), which is used to evaluate the colon, is not indicated in this case.

 

Explanation:

The correct answer is A. The patient is presenting with classic signs and symptoms of renal colic. Ureteral calculi are a major cause of acute urinary tract obstruction and renal colic. When calculi are passing through the ureters, they can cause severe flank pain and hematuria. Most (up to 80%) of renal calculi can be seen on plain abdominal radiographs. When not seen in the initial study, the next best step is a CT of the abdomen and pelvis. A CT is very useful because it can identify these dense structures within the collecting system as well as give information about urinary tract obstruction.

Urine culture (choice B) is not indicated in this patient as she does not have signs or symptoms of a urinary tract infection. Flank pain can be a symptom of pyelonephritis. She does not, however, have other signs and symptoms of a urinary tract infection such as fever, dysuria, frequency and urgency with urination, and white blood cells in a urinalysis.

Giving the patient pain medications only (choice C) is not appropriate. Although she does have classic signs and symptoms of renal colic, a diagnostic study should still be performed to confirm the diagnosis, as well as to evaluate for other possible etiologies. Most patients with small renal and ureteral calculi are simply treated with pain medications because the calculi will pass on their own. However, larger obstructing stones may require intervention in addition to pain medications.

Plain radiographs of the thoracic and lumbar spine (choice D) are not indicated in this patient. Flank pain is most often associated with the urinary tract as in renal colic and pyelonephritis. Flank pain, even with radiation down the back, is not associated with bony abnormalities of the spine.

Renal ultrasound (choice E) is most useful for visualizing large stones (>1.5 cm) and small stones are often not demonstrated. Although renal ultrasound is more sensitive than plain abdominal radiographs for detecting subtle calcifications, it is not as sensitive as CT.

 

  A 62-year-old man with a long history of cigarette smoking comes to the office with a 3-month history of painless gross hematuria. Physical examination is unremarkable. Urologic evaluation, including cystoscopy, reveals a medium-sized bladder tumor. You recommend a surgeon to the patient and a transurethral resection is performed. The pathology shows high-grade transitional cell carcinoma invading the muscularis propria. A metastatic workup is negative and the patient is counseled regarding radical cystectomy and urinary diversion. A radical cystectomy, pelvic lymph node dissection, and ileal conduit are performed successfully. The surgical margins and lymph nodes are all negative. An 18-month follow-up CT scan of the pelvis reveals a 4-cm heterogeneous, contrast enhancing mass. A biopsy shows a high-grade transitional cell carcinoma. The patient is referred to an oncologist who suggests chemotherapy using a platinum-based regimen. He comes back to your office and tells you that he has heard so many “horror stories” about chemotherapy and that he is concerned about the toxic side effects of the recommended platinum-based regimen. He should be told that this regimen significantly increases his risk of developing

  A. cardiac toxicity
  B. myelosuppression
  C. nephrotoxicity
  D. neurotoxicity
  E. pulmonary toxicity
Explanation:

The correct answer is C. The mainstay of treatment for advanced, metastatic, or recurrent urothelial carcinoma involves platinum based chemotherapy regimens, usually in combination regimens. The most commonly used regimen is MVAC using methotrexate, vinblastine, adriamycin, and cisplatinum. Unfortunately, toxicity often limits the usefulness of these regimens. The toxicity most often associated with platinum is nephrotoxicity. Acute tubular necrosis develops in approximately 25% of patients, and is often the dose-limiting factor. This toxicity can be prevented by keeping patients well hydrated and using diuretics during therapy.

Cardiac toxicity (choice A) is associated with doxorubicin (adriamycin). A cumulative dose-related cardiomyopathy results from doxorubicin treatment and can be fatal.

Myelosuppression (choice B) is an adverse affect of many chemotherapeutic agents. Most noteworthy are methotrexate, vinblastine, and doxorubicin.

Neurotoxicity (choice D) is associated with the vinca alkaloids, especially vincristine. Vincristine can produce a dose-related mixed motor-sensory and autonomic neuropathy.

Pulmonary toxicity (choice E) is associated with bleomycin. Pulmonary fibrosis can result from bleomycin and pulmonary function tests may be necessary in these patients.

 

  A 68-year-old woman comes to the clinic complaining of “a mass in her vagina”. She noticed the mass a couple years ago, but did not think much of it because it caused her no pain. However, over the last 6 months she noticed that the mass is increasing in size. The patient tells you that the mass is more prominent when she is standing, and there has been some difficulty in emptying her bladder, for which she has learned to “push the mass back in” so that she can void completely. She also complains of dyspareunia over the past 6 months as well. She denies any urinary tract infection, renal stones, difficulty with ambulation, trauma to the area, or vaginal surgery. She has 5 grown children, all born vaginally. On physical examination, her abdomen is soft and nontender, without organomegaly or palpable masses. On pelvic examination, the vaginal mucosa is atrophic and dry. The vaginal vault is completely occupied by a smooth round mass that protrudes beyond the introitus. The mass is nontender, soft, and easily reduced anteriorly so that the vaginal vault may be completely examined. With the mass reduced, the urethral meatus is grossly normal in appearance. The cervix is without lesions or tenderness. No adnexal masses are appreciated. With the mass protruding, you ask the patient to cough which causes further protrusion and leakage of some urine. When you manually reduce the mass and ask the patient to cough, there is no loss of urine. Urinalysis is normal. The most likely diagnosis of this “mass” is

  A. cystocele
  B. paraurethral cyst
  C. sarcoma botryoides of the vagina
  D. transitional cell carcinoma of the bladder
  E. urethral prolapse
Explanation:

The correct answer is A. This woman is suffering from a grade IV cystocele. A cystocele is present when the bladder base descends below the inferior ramus of the symphysis either at rest or straining. They are graded on a scale of I (minor) to IV (severe, with bladder base outside the vaginal introitus at rest). Cystoceles occur due to the development or progression of pelvic prolapse. This is frequently seen with relaxation of the pelvic floor following childbirth. With a cystocele, pelvic examination usually reveals relaxation of the anterior vaginal wall and descent of the urethra and bladder, with worsening when the patient strains to void. Complete examination should include manual reduction of the cystocele to evaluate the bladder for stress incontinence, which can occur after surgery to repair a cystocele. Thorough evaluation includes history and physical, urine culture, video-urodynamics or a voiding cystourethrogram (VCUG). In severe cases a renal ultrasound should be performed to rule out silent hydronephrosis secondary to ureteral obstruction. While less severe cases may be observed, patients with large cystoceles or severe symptoms may require surgery to recreate the anterior vaginal wall support.

A paraurethral cyst (choice B) occurs in the newborn. It results from retained secretions in the Skene’s glands secondary to ductal obstruction. The cyst tends to displace the urethral meatus. Typically, these cysts regress over the first 4-8 weeks of life.

Sarcoma botryoides (choice C) is a type of sarcoma that is found in the bladder or vagina. It has a classic appearance of a cluster of grapes. This tumor commonly occurs in the pediatric age group.

Transitional cell carcinoma of the bladder (choice D) occurs inside the bladder. It may be asymptomatic, other than hematuria, microscopic or gross. It does occur in this age group. If there is extension beyond the bladder wall, this usually appears as a fixed mass on pelvic exam, not a smooth, soft, reducible mass.

Urethral prolapse (choice E) has the appearance of an erythematous, inflamed protruding mucosa surrounding the urethral meatus. It occurs almost exclusively in black girls between 1 and 9 years of age. The most common signs are bloody spotting on the underwear or diaper. Initial therapy is conservative management with application of estrogen cream.

 

  You get a call from a 27-year-old woman complaining of a urinary tract infection. She tells you that she has had burning on urination and frequency for the past 2 days. She says that she took a urinary tract infection detection test that she bought at the pharmacy, and that it is positive for nitrites. She wants you to prescribe antibiotics for her. You ask your nurse to pull out this patient’s chart, but she cannot find it. You return to the phone and ask the patient when was the last time she was in to see you. She tells you that she recently moved to your town and has not yet seen a physician in the area. A colleague in her office gave her your name and number but she has not had a chance to schedule an appointment. She is generally very healthy, has no chronic medical conditions, does not take any medications, and has no known drug allergies. She is very busy at work and would like you to either prescribe an antibiotic over the phone or have a prescription ready for her to pick up, and drop off at the pharmacy. At this time you should

  A. advise her to void soon after intercourse and to avoid spermicide use
  B. prescribe a 3-day course of trimethoprim-sulfamethoxazole
  C. recommend that she schedule an appointment and that you will evaluate her for the appropriate treatment
  D. recommend that she take blueberry tablets and drink cranberry juice to treat her symptoms
  E. tell her that over-the-counter urinary tract detection tests for nitrites are not accurate
Explanation:

The correct answer is C. Since this patient has never been in to see you before and you have absolutely no medical records for her, it is inappropriate to treat her over the phone and to prescribe medication. You should evaluate this patient in the office and determine the appropriate treatment based on the history and physical findings and consider additional diagnostic testing (urinalysis, microscopy, or culture and sensitivity).

Since you have never seen this patient and have no past medical history or chart on her, you should have her come into the office for an evaluation before you make any suggestions on how to prevent UTIs in the future, such as advising her to void soon after intercourse and to avoid spermicide use (choice A) or on how to treat the symptoms (choice D). If she truly has an infection, you can provide treatment and then make recommendations on methods to prevent future infections and on reducing symptoms. Some believe that blueberries and cranberries contain compounds, such as tannins and pyoanthocyanidins, that may prevent the adherence of the pathogen to the urinary tract lining. They may be useful in helping prevent infections, but an antibiotic is usually necessary to treat an established infection.

Prescribing a 3-day course of trimethoprim-sulfamethoxazole (choice B) without ever seeing this patient is inappropriate. Sometimes it is difficult to deal with pushy patients who want antibiotics without being examined, but this is a clear-cut case where she should come in for an examination because she is not an established patient.

It is incorrect to tell her that over-the-counter urinary tract detection tests for nitrites are not accurate (choice E). These tests detect nitrites, which are released as part of the metabolic process of certain UTI-causing bacteria. These tests, if performed according to the directions, are generally accurate for detecting nitrites in the urine.

 

A 78-year-old woman is admitted to the hospital because of fever, flank pain, and mental obtundation. On arrival to the hospital, she was minimally responsive and was found to have a white blood cell count of 43,000/mm3 with a profound left shift. Urinalysis revealed packed white cells. A renal ultrasound demonstrated a left hydronephrosis and hydroureter. A CT scan confirmed the presence of an obstructing stone. Her vital signs on admission showed a temperature of 39.5 C (103.1 F), blood pressure of 80/40 mm Hg, pulse of 112/min, and respirations of 18/min. Intravenous pressors were initiated. Three sets of blood cultures came back positive for Gram-negative rods within 2 hours. The most appropriate next step in management is to

  A. administer antibiotics, intravenously, and observe
  B. insert a nephrostomy tube, percutaneously
  C. perform extracorporeal shock wave lithotripsy (ESWL)
  D. schedule cystoscopy and ureteral stent placement
  E. schedule a nephrectomy
Explanation:

The correct answer is B. This patient has obstructing pyelonephritis with sepsis syndrome. She is critically ill and requires immediate decompression of her kidney and removal of the infected urine. The most expeditious and least invasive way to accomplish this is percutaneous introduction of a nephrostomy tube under local anesthesia. There will be no need for general anesthesia and the procedure is low risk for a patient already gravely ill.

Since this patient is gravely ill from an obstruction in her ureter, intravenous antibiotics and observation (choice A), although addressing the sepsis fails to address decompression. This patient will die if the stone is not removed.

Extracorporeal shock wave lithotripsy (ESWL) (choice C) is an elective procedure for reducing large stones to smaller ones so that they can be passed naturally. This is not appropriate for emergency situations.

A cystoscopy and ureteral stent placement (choice D) will be ineffective at removing the obstructing stone and will also require general anesthesia since access of the ureter is highly stimulating (unlike a cystoscopy which can be done in the office).

Since the kidney is not itself diseased, there is no indication for nephrectomy (choice E). This procedure is most often performed for tumor removal.

 

 

Explanation:

The correct answer is B. The management of hematuria associated with trauma differs in adults and children. In the adult population, imaging is performed only in those patients with gross hematuria or microscopic hematuria plus hypotension. This differs from the pediatric patient. In children, any degree of hematuria (gross or microscopic) should be investigated with imaging studies. One reason for this discrepancy is that large amounts of catecholamines released in injured children may sustain blood pressure in the face of hypovolemia. A CT scan is the most useful imaging modality in this setting. A CT scan is noninvasive, accurate and fast, and it can help in assessing the size and extent of retroperitoneal hematomas and renal parenchymal trauma. Not only does this child have microscopic hematuria (an indication by itself to perform imaging studies), but he also has signs, (flank ecchymosis and tenderness), that raise the suspicion of renal injury. High suspicion for renal injury (i.e., rib fracture, flank contusion, deceleration injury) is another indication for perform imaging studies.

There is no indication at this time that the patient requires antibiotics (choice A). He has no open fractures, large abrasions, or burns. Further diagnostic studies must be undertaken to determine if the patient requires antibiotics for any disruption to the urinary system.

Ordering a renal/bladder ultrasound (choice C) acknowledges the fact that the patient does require investigation of his urinary system. However, a CT scan is quicker and will have a much higher yield for associated urinary and abdominal injuries versus ultrasound.

A retrograde urethrogram (choice D) allows for visualization of the urethra to investigate for extravasation. It is performed when there is gross blood at the meatus. It will not help in this patient whose injuries are suspected to be intraabdominal.

A hematuria is not an indication for Foley catheter placement (choice E). As long as the patient is awake without altered sensorium, he should be given an opportunity to void on his own (which he has done). It should be noted that in any patient who has gross blood at the meatus, a retrograde urethrogram must be obtained prior to placing a Foley catheter.

Repeating the urinalysis (choice F) will only delay this patient’s workup. Although the urine dipstick may be falsely positive for heme, there is no reason to doubt the validity of the urinalysis. Besides, this patient’s flank ecchymosis and tenderness warrant imaging independently of his urinalysis results.

An intravenous pyelogram, or IVP, (choice G) has a role in evaluating the urinary tract. However, with the use of a CT scan, IVP has a very limited role in evaluating renal/ureteral trauma. Delaying imaging until the patient is an outpatient may miss a potentially life threatening renal injury.

 

A 94-year-old man is transferred from his nursing home to the hospital because of an altered mental status over the past 10 days and little oral intake. His past medical history is significant for diabetes mellitus, coronary artery disease with 2 prior myocardial infarctions, congestive heart failure with an ejection fraction of 20%, and chronic renal insufficiency. He was diagnosed with a renal-cell carcinoma metastatic to the brain, lungs, and liver 1 month ago. His temperature is 37.8 C (100 F), blood pressure is 89/54 mm Hg, pulse is 128/min, and respirations are 36/min. His heart is tachycardic with a 2/6 systolic ejection murmur, lungs have coarse breath sounds bilaterally, and his abdominal examination is benign. It is felt that the patient will require endotracheal intubation in order to survive. However, the patient is transferred with a living will that was written up after his diagnosis with renal-cell carcinoma, which states that he does not want any extraordinary measures taken to prolong his life. Before a do not resuscitate (DNR) order is written, his son arrives and says that everything medically possible should be done in order to save his father’s life. The most appropriate management at this time is to

  A. call a family meeting and discuss with the son and any other family members why he or they would like to override the wishes of his father with regard to his terminal care
  B. call the nursing home and have the patient transferred back
  C. intubate the patient and admit him to the intensive care unit
  D. politely ask the son to leave the room and start the patient on a morphine drip to keep him comfortable in his final hours
  E. tell his son that there is no way you are going to listen to him or do anything to prolong the patient’s life beyond the wishes set forth in the living will, then transfer the patient to hospice care
Explanation:

The correct answer is A. When a patient can no longer speak for himself or herself, it is always a very difficult ethical dilemma, when a close family member wants to override the wishes set forth by a patient in a living will. The best course of action is to get the appropriate parties together to have a logical discussion about why or why not, the wishes of the family member should be honored.. The original decision is often better thought out at a time of less emotional stress. Whereas, at a time when a patient may die, the family will often change their decisions about care, when the fear of losing a loved one is staring them in the face.

What will end up transpiring after a logical and calm discussion with the family will often vary. It is always desirable to honor the wishes of a living will. However, you never know when extenuating circumstances may exist. It is important to treat the patient and the family with the utmost respect. Telling the son that you won’t listen to him (choice E), is inappropriate. Similarly asking the son to leave the room (choice D), politely or not, is also not appropriate.

Transferring the patient back to the nursing home (choice B) or admitting him to hospice, and even to a certain extent, after discussions, intubating the patient (choice C) may all end up being the final course of action.

 

A 21-year-old woman comes to the clinic because of painful urination. She also reports that her urine has progressively turned from pink to red over the past 2 days. She has never had symptoms like this before. She is sexually active with 1 partner and they use condoms for birth control. She has a past medical history of depression treated with psychotherapy. Vital signs are: temperature 38.9 C (101.9 F), blood pressure 100/70 mm Hg, pulse 102/min, and respirations 12/min. Physical examination reveals left-sided costovertebral angle tenderness. The pelvic exam is unremarkable. Laboratory studies show:

The next step in the management of this patient should be

  A. a blood culture and spinal fluid analysis
  B. a blood culture and urinalysis
  C. an MRI of the lumbar spine
  D. an ultrasonography of the kidneys
  E. an x-ray of the lumbar spine
Explanation:

The correct answer is B. Blood cultures and urinalysis are necessary because this patient has symptoms of pyelonephritis. The presentation of urinary symptoms, fever, leukocytosis, and bandemia is clinically diagnostic of pyelonephritis. A urine culture should also be obtained prior to starting antibiotics. Empiric broad spectrum antibiotics should be started and coverage can be narrowed when culture and sensitivity data are available. A renal ultrasound can also be performed at a later time to evaluate the kidneys if the patient does not respond to antibiotic therapy. The renal ultrasound is useful to evaluate for the complications of pyelonephritis such as abscess or obstruction.

This patient has no mental status changes or headaches that would necessitate a lumbar puncture (choice A)

This patient has no radiculopathy or back pain that would necessitate an MRI of the lumbar spine (choice C). Costovertebral or flank tenderness is often a sign of kidney inflammation or infection.

An ultrasonography of the kidneys (choice D) is not necessary at this time. A renal ultrasound can be performed if the patient does not respond to antibiotic therapy. The ultrasound is useful to evaluate for complications of pyelonephritis such as abscess or obstruction.

The physical examination of the back reveals flank, not lumbar spine pain. Hence, an x-ray of the lumbar spine (choice E) is not necessary.

 

A 61-year-old woman comes to the office because of lower and upper extremity swelling. She has a long history of hypertension, hyperlipidemia, and gout that have been very well controlled. She is an active woman who works as a fashion store manager. She takes thiazide, mevastatin, and allopurinol daily. Over the past few weeks, she has noticed increasing swelling of her feet and her hands. Her feet have gotten so swollen that this morning she was unable to put her shoes on. Her temperature is 37 C (98.6 F), blood pressure is 180/70 mm Hg, pulse is 72/min, and respirations are 12/min. Blood chemistries are remarkable for a BUN of 40 mg/dL and a creatinine of 1.8 mg/dL. A urine dipstick is positive for protein. A 24-hour urine test confirms 4gm of protein. The most important intervention at this time is to

  A. add a loop diuretic
  B. increase thiazide dosage
  C. initiate ACE inhibitor therapy
  D. recommend a high protein diet
  E. start hemodialysis
Explanation:

The correct answer is C. The nephrotic syndrome is defined by a urinary protein level exceeding 3.5 g per 1.73 m2 of body-surface area per day. Diabetic nephropathy is the most common cause of nephrotic proteinuria. Five primary glomerular diseases account for the great majority of cases of the nephrotic syndrome in persons who do not have diabetes. In adults, the most common cause is membranous glomerulonephropathy. A common clinical triad of the nephrotic syndrome is hypertension, hyperlipidemia, and proteinuria. Although the exact mechanism whereby edema formation occurs in these patients is uncertain, the loss of urinary protein leads to total body edema formation. Regardless of the magnitude of the urinary protein loss, initiating ACE inhibitor therapy has been shown to be beneficial in terms of both decreasing the urinary protein content and prolonging survival.

Adding a loop diuretic (choice A) or increasing the thiazide dosage (choice B) fails to address the etiology of the edema. This patient has new onset edema and simply trying to manage the symptom by increasing an existing diuretic dosage or adding a second diuretic class agent fails to address the underlying etiology of this patient’s edema.

There is no role for a high-protein diet (choice D) in managing these patients. Contrary to previously held opinions that the protein needed to be replaced by high-protein diets, it is now clear that such diets exacerbate kidney damage and accelerate protein loss. Low-protein diets are recommended for these patients.

There is no indication for hemodialysis (choice E) at this time. The five indications for HD are refractory hyperkalemia, volume overload, acidosis, uremia, or uremic pericarditis.

 

You admit a 46-year-old woman, who is your medical partner’s patient, to the hospital for the evaluation of an acute deterioration in renal function. She was seen by your partner, who is now on vacation, 2 days earlier for some mild upper respiratory complaints including sore throat, cough, and fever. He prescribed cephalexin and sent her home. Today, as you are reviewing all of the laboratory data that returned in the past couple of days, you notice that this patient has a blood urea nitrogen level of 67 mg/dL and a creatinine level of 2.1 mg/dL. You call her and tell her to meet you at the hospital for further evaluation. On admission her blood urea nitrogen level is 109 mg/dL and creatinine level is 4.2 mg/dL. The most appropriate study to order at this time is

  A. serum electrolytes, including phosphate and calcium
  B. serum lipid profile
  C. 24-hour urine for protein and electrolytes
  D. urine for sediment analysis
  E. urine eosinophils
Explanation:

The correct answer is A. In any case of rapidly deteriorating renal function, the substances normally cleared and regulated by the kidneys will be acutely deranged and must be evaluated. This step is critical because irrespective of the etiology of the renal failure, these abnormalities will be present. The findings of hyperkalemia, hyperphosphatemia, acidosis, or uremia will prompt immediate intervention prior to any diagnostic evaluation being undertaken.

Serum lipid profile (choice B) is not a useful test in the urgent evaluation of acute liver failure. It is commonly ordered when the diagnosis of nephrotic syndrome is suspected.

Twenty-four hour urine for protein and electrolytes (choice C) is a non-emergent test used to quantify the amount of protein and other substances being lost by the kidney each day.

Urine for sediment analysis (choice D) is incorrect. This patient has rapidly declining renal function. What is critical in the evaluation of all renal insufficiency, especially rapidly progressive disease, is the examination of the sediment for the presence of casts. It does not however take precedence of the determination of serum K, phosphate, and acid load.

Urine eosinophils (choice E) is a non-emergent test most often ordered when drug-induced acute interstitial nephritis (AIN) is suspected.

 

You are seeing a 62-year-old woman with diabetes, hypertension, and depression in the emergency department who is complaining of feeling “ill”. Her medications include NPH insulin, furosemide, captopril, and sertraline. She denies any alcohol or tobacco use. Her temperature is 37.0 C (98.6 F), blood pressure is 98/60 mm Hg, pulse is 102/min, and respiratory rate is 24/min. Her lungs are clear to auscultation and cardiac rhythm is regular. No rubs are audible. A chest radiograph shows no pathology. An electrocardiogram shows sinus tachycardia. You send laboratory studies and find that her serum creatinine is 4.0 mg/dl, up from its baseline of 1.0 mg/dl 4 days ago. Her blood glucose is 100 mg/dl. The most appropriate next step is to evaluate her urine for

  A. glucose level
  B. potassium level
  C. protein level
  D. sodium level
  E. toxicology screen
Explanation:

The correct answer is D. The basic differential diagnosis for acute renal failure is a prerenal versus a postrenal versus an intrinsic renal etiology. The easiest etiologies to rule out are the prerenal and postrenal causes of acute renal failure. Prerenal causes of acute renal failure are associated with hypovolemia. Therefore, these etiologies are typically associated with sodium avid states as the kidney fights to retain volume. A urine sodium in the setting will be less than 25 mmol/L.

Sending urine for a glucose level (choice A) will be of no benefit since it will not provide any insight into the cause of her acute renal failure. One would also not expect any glucosuria since the patient is normoglycemic.

Sending urine for potassium (choice B) does not provide insight into the cause of acute renal failure. Looking for hyperkalemia in the setting of renal disease is however reasonable given the morbidity associated with hyperkalemia.

Sending urine for a protein level (choice C) does not provide insight into the cause of acute renal failure. However, proteinuria or microalbuminuria in this patient will indicate the presence of diabetic nephropathy which places patients at higher risk for renal failure.

Sending urine for a toxicology screen (choice E) will be of no benefit since we are not given any history to suggest toxins as the cause of this patient’s acute renal disease.

 

A 43-year-old man comes to the emergency department with bilateral lower back pain. He has had a dull pain for the last 3 days, and today noticed that his urine had a pink tinge. He has no significant past medical history and takes no medications. He denies fever or any other symptoms. Vital signs are: temperature 37.0 C (98.6 F), blood pressure 120/67 mm Hg, and pulse 70/min. Physical examination reveals bilateral costovertebral angle tenderness. Urinalysis shows:

A CT scan of the abdomen and pelvis is performed and a single image from the study is shown.

The most appropriate management of this patient is to

  A. order an emergent intravenous pyelogram (IVP)
  B. order an emergent renal ultrasound
  C. prescribe antibiotics and schedule an appointment in an outpatient clinic
  D. prescribe pain medications and schedule an appointment in an outpatient clinic
  E. reassure the patient that CT scan is normal
Explanation:

The correct answer is D. The CT of the abdomen demonstrates multiple cysts in both the kidneys and the liver. This is a classic finding of autosomal dominant polycystic kidney disease or ADPKD. The most common symptoms are headaches and dull pain in the back and the flank. Other associated findings or presentations include urinary tract infections, hematuria, liver and pancreatic cysts, and high blood pressure. There is no cure for ADPKD and the treatment consists of preventative measures and symptomatic treatment. This patient’s only symptom is pain. The appropriate treatment is pain management and referral to outpatient care for the monitoring of blood pressure and renal function.

An emergent intravenous pyelogram (IVP) or renal ultrasound (choice A and B) are not necessary. The diagnosis of autosomal dominant polycystic kidney disease (ADPKD) can be made by the classic findings on the CT. Pain management and outpatient referral are the most appropriate next steps.

Antibiotics (choice C) are not necessary. The patient has no signs of urinary tract infection such as fever, dysuria, or pyuria. It is appropriate to treat his pain and to refer him to an outpatient clinic for monitoring of blood pressure and renal function.

Reassuring the patient that the CT scan is normal (choice E) is inaccurate. Autosomal dominant polycystic kidney disease is a genetic disorder that eventually leads to end-stage renal disease. There are preventative measures to prolong life and treat symptoms, and it is essential that the patient have outpatient follow-up care particularly for monitoring of blood pressure and renal function.

 

A 37-year-old comes to the clinic for a required pre-employment physical examination. She has no past medical history and has no complaints. Her temperature is 37.0 C (98.6 F), blood pressure is 110/60 mm Hg, pulse is 63/min, and respirations are 14/min. She is currently menstruating. A urine culture, which is required by her new job, reveals greater than 100,000 E. Coli colony-forming units. The most appropriate next step in management of this patient’s urine culture findings is

  A. no further management is indicated
  B. prescribe ciprofloxacin, orally, for 14 days
  C. prescribe trimethoprim-sulfamethoxazole, orally, for 3 days
  D. repeat urine culture in 2 weeks
  E. send her for urological evaluation
Explanation:

The correct answer is A. This patient has asymptomatic bacteriuria. This is a common condition in which women with no symptoms of urinary tract infections have a urine culture which grows a significant number a bacteria. It is not necessary to treat patients with asymptomatic bacteriuria unless they have an underlying structural or functional abnormality of their kidney or they are pregnant.

Ciprofloxacin treatment for 14 days (choice B) is indicated for patients with pyleonephritis. These are the patients who present with fevers, chills, flank pain, and lower urinary tract symptoms. They can also have positive blood cultures in addition to urine cultures. More intensive workup of the etiology of this infection can be pursued.

Trimethoprim and sulfamethoxazole (choice C) for 3 days is appropriate treatment for women with an uncomplicated urinary tract infection. These are patients with a significant bacterial count of greater than 105 organism/mL of urine, pyuria, and positive irritative voiding symptoms.

Repeat urine cultures (choice D) are unnecessary in asymptomatic patients with bacteruria. Repeat urine cultures are also not necessary in patients with uncomplicated urinary tract infections.

Urological evaluation (choice E) is reserved for those patient with recurrent infections, treatment failures, and those with pyleonephritis.

 

A 72-year-old man is admitted to the hospital because of nausea and vomiting for several days. He states that he has been urinating, but it has been decreased in volume from his baseline. He has also had some mild lower abdominal pain. His past medical history is significant for coronary artery disease, mild emphysema, cholelithiasis, and benign prostatic hypertrophy. His temperature is 37.2 C (99 F), blood pressure 149/88 mm Hg, pulse 72/min, and respirations 18/min. His heart is regular with no murmurs or rubs and his lungs have some scattered wheezes. His abdomen is soft, with normal bowel sounds, and there is the suggestion of a mass in the suprapubic region. Laboratory studies show a leukocyte count 6,800mm3, platelets 218,000mm3, hematocrit 39%, BUN 72 mEq/L, creatinine 3.2 mEq/L, sodium 138 mEq/l, and potassium 4.9 mEq/l. The next most appropriate step in management is to

  A. administer a bolus of 1 L of normal saline and then half normal saline at 100 ml/hour
  B. give no treatment at this time
  C. place a Foley catheter and keep it to drainage
  D. provide prednisone 60 mg intravenously then follow with a steroid taper
  E. order a renal ultrasound to rule out obstruction
Explanation:

The correct answer is C. In any older man presenting with acute renal failure, particularly one with a history of prostatic hypertrophy, the possibility of urinary obstruction as a cause of renal failure should be the prime diagnostic consideration. This patient also has a suprapubic “mass” which represents a markedly enlarged bladder. His nausea and vomiting is likely due to the renal failure. The easiest way to, not only diagnose, but to treat this entity is placement of a Foley catheter to relieve the obstruction. It is important to note that even though this patient has a high-grade blockage he still is producing urine likely due to overflow from enough increased pressure in the bladder to overcome the obstruction. Thus, the presence of urine output should not dissuade you from placing a bladder catheter.

Giving a bolus of normal saline (choice A) is used for treatment in patients with prerenal volume depleted states causing acute renal failure. These patients often have a history of volume depletion and other laboratory values or physical findings suggestive of this. A BUN to creatinine ratio is often helpful to sort out the cause of the renal insufficiency. A ratio of greater than 20:1 often implies pre-renal dysfunction, and a ratio closer to 10:1 often implies intrinsic renal disease. Postrenal dysfunction can present with a variety of ratios.

No treatment (choice B) is not an appropriate choice.

Prednisone (choice D) is an immunosuppressive agent and is often used in renal failure caused by autoimmune diseases. This clinical scenario is much more consistent with postrenal obstruction.

A renal ultrasound (choice E) is commonly used in the evaluation of urinary obstruction. However, it is only a diagnostic maneuver and may be of limited availability, particularly late at night. Placement of a catheter as mentioned above is not only diagnostic, but also therapeutic, and is easy to perform. It should generally be performed prior to an ultrasound.

 

  A 40-year-old man is rushed into your emergency department. He was the restrained passenger in a motor vehicle accident involved in a hit and run. Upon arrival, he is awake and communicating. He denies any loss of consciousness and his only complaint is right-sided abdominal pain. As the physician in charge of the case, you assign your team to assess the patient completely. He is evaluated by standard trauma protocol. His vital signs are stable, and his neck and spine are determined to be without injury. Upon examination of his abdomen, you elicit a moderate amount of tenderness in the right upper quadrant. There is associated guarding, but no rebound. There is no flank ecchymosis or clinical suspicion of rib fractures. No abnormalities are noted on genital examination and no extremity fractures are appreciated. His serum chemistries and complete blood count are within normal limits. However, urinalysis (from a spontaneously voided specimen) is significant for 35 RBC, 3 WBC, negative leukocyte esterase, and negative nitrates. You are concerned about his right upper quadrant pain and elect to perform a CT scan. A CT scan of the abdomen and pelvis reveals a subcapsular hematoma with a superficial laceration of the right renal cortex. There is no extravasation of intravenous contrast from the right kidney. No free air is seen and no other organs appear to have any damage. The most appropriate way to manage this patient’s renal injury is to

  A. admit the patient for serial abdominal exams and blood counts
  B. obtain an intravenous pyelogram
  C. order a retrograde urethrogram
  D. perform angiography of the right kidney
  E. place a Foley catheter and perform bladder irrigation
  F. surgically explore and evacuate hematoma
Explanation:

The correct answer is A. This patient has a grade II renal injury. The kidney is the most common organ injured in the urinary system, with most injuries occurring from automobile accidents or sporting mishaps. Kidneys with pathological conditions such as hydronephrosis or malignant tumors are more readily ruptured from mild trauma. Blunt trauma is the most common mechanism of renal injury, causing 80-85% of all renal trauma. Gunshot and knife wounds are responsible for most penetrating renal trauma.Renal trauma can be classified as minor, major, and vascular and are rated from a grade I up to grade V. For minor injuries, a renal contusion of the parenchyma is the most common lesion, and usually associated with a subcapsular hematoma. A superficial cortical laceration may also be noted. These injuries rarely require surgical exploration. Major renal trauma occurs when lacerations extend deeply into the corticomedullary and collecting system tissue. This results in extravasation of urine into the perirenal space. Many of these injuries can also be managed conservatively, however, it is not uncommon to perform surgical exploration of the most serious injuries from this category. Vascular injuries to the renal pedicle are rare. It may be caused by total avulsion of the artery and vein or thrombosis secondary to stretching of the main renal artery. Vascular injuries are difficult to diagnosis and frequently result in total kidney destruction.

Intravenous pyelogram, or IVP, (choice B) is performed to establish the presence or absence of both kidneys, to define renal outlines and cortical borders, and to outline the collecting systems and ureters. All of these goals are reached with the CT scan you ordered for this patient. Therefore, an IVP is not indicated at this time. Prior to a CT scan, it was the method of choice for initial radiological imaging of kidneys suspected of traumatic injury.

A retrograde urethrogram (choice C) is performed in patients suspected of having a urethral injury. These patients typically have blood at the meatus, a perineal hematoma, a high riding prostate, a pelvic fracture, or a mechanism of injury suspicious for urethral damage. This patient does not meet any of these criteria. He also voided spontaneously, which is frequently not the case with urethral injury. His microscopic hematuria is from his renal injury.

Angiography (choice D) can be used to define the major arterial and parenchymal injuries of the kidney. In patients who are actively bleeding, this exam will also allow for embolization of a lacerated blood vessel. In this patient with a minor renal injury and stable vital signs, it is not indicated at this initial stage.

This patient has voided spontaneously with grossly clear urine. His hematuria is microscopic. There is no evidence that he has any bladder injury or that he is suffering from clot retention within his bladder. Therefore, there is no need to place a catheter or to irrigate his bladder (choice E).

Surgical exploration (choice F) is not indicated in a patient who is hemodynamically stable, has no other organ injury, and does not appear to have any evidence of continued bleeding. Attempting to evacuate this small hematoma is not necessary as this minor injury will most definitely remain stable and resolve on its own. For this reason, admitting the patient for serial evaluations is the most appropriate step in management. If the patient’s injury continues to progress, it will manifest itself clinically with dropping hematocrit, worsening abdominal pain, and hemodynamic instability.

 

A 31-year-old previously healthy man comes to your office because of a 2-week history of low-grade fevers, weight loss, malaise, nocturnal tightness in his chest, and shortness of breath. He also reports a small amount of leg swelling and scrotal swelling over this time. Prior to 2 weeks ago, he denies any recent illnesses, sick contacts, or travel. He also denies any hemoptysis or sinus infections. His temperature is 38.0 C (100.2 F), blood pressure is 170/95 mm Hg, pulse is 77/min, and respirations are 14/min. Physical examination shows trace bilateral lower extremity and scrotal edema. Bilateral wheezes are also appreciated. A chest x-ray is unremarkable. Laboratory studies show:

Sodium 141 mEq/dL
Potassium 5.1 mEq/dL
Chloride 98 mEq/dL
Bicarbonate 21 mEq/dL
Urea nitrogen, serum 21 mg/dL
Creatinine 2.5 mg/dL
Glucose 97 mg/dL
Serum eosinophils mildly elevated

 

Urinalysis
Color clear
Specific gravity 1.020
Osmolality 55 mOsmol/kg
Leukocyte esterase negative
Nitrite negative
Protein 2+
Blood 4+
Microscopic many red cell casts.
24 hour urine protein collection 1900 mg

The laboratory finding that would support the most likely diagnosis is

  A. anti-glomerular basement membrane antibody
  B. antistreptolysin titer (ASO)
  C. cANCA
  D. elevated serum IgA levels
  E. pANCA
Explanation:

The correct answer is E. This patient likely has Chrug-Struass syndrome. Chrug-Struass syndrome is a nephritic syndrome, which is associated with eosinophilia and asthma. This patient has nephritic syndrome as evidenced by edema, hypertension, and hematuria. Asthma is suggested by his nocturnal chest tightness and shortness of breath. Laboratory results reveal elevated eosinophils, which is typical in this condition. Chrug-Strauss is associated with a positive pANCA.

Goodpastures syndrome is typically defined as a combination of nephritic syndrome and pulmonary hemorrhage, although some patients have nephritic syndrome alone. The disease is mediated by antibodies to the glomerular basement membrane (choice A) and therefore those are the laboratory results often associated with the disease.

ASO (choice B) is associated with post infectious glomerulonephritis. Patients present with oliguria, hypertension, and edema. They classically complain of Coca-Cola colored urine. These infections classically occur 1-3 weeks after infection with nephritogenic group A streptococci (step throat or cellulitis).

Wegner granulomatosis is similar to Chrug-Strauss but respiratory involvement such as nasal septal perforation or sinus problems are typical presenting signs. Wegners doesn’t have an association with asthma or increased serum eosinophils. The test associated with Wegners is cANCA (choice C).

Bergers syndrome (IgA nephropathy) presents in a similar manner to post-infectious glomerulonephritis but there is no latent period between infection and kidney involvement. These patients typically have gross hematuria after a viral illness. About 50% of these patients will have an elevated IgA (choice D).

 

A 76-year-old man comes to the clinic because of “urine problems”. He tells you that he has “trouble with his urinary stream,” that over the past month it has been progressively decreasing in force. He also has the urgency to urinate and he finds himself running to the bathroom, but can only pass a small amount of urine. The trips to the bathroom have increased in frequency over the past couple of days and he has started to leak urine. The leakage is only in small amounts, but can occur at any time. Yesterday, he ruined a new pair of pants and is visibly upset and embarrassed. He has hypertension for which he takes a calcium channel blocker and he underwent a laparoscopic cholecystectomy a few months ago. His abdominal examination reveals a smooth, round suprapubic mass. Percussion of this mass is dull and causes the patient to leak urine per urethra. Penile examination is normal. Rectal examination reveals good sphincter tone with a moderately enlarged, smooth prostate (also unchanged from prior exams). Urine dipstick in the office is negative for glucose, RBC, WBC, and nitrites. You explain to the patient that the most likely primary cause of his incontinence is a/an

  A. abnormal bladder sensation
  B. obstructing prostate
  C. uninhibited bladder contractions
  D. vesicocutaneous fistula
  E. weakened pelvic floor muscles
Explanation:

The correct answer is B. The patient has overflow incontinence. The suprapubic mass is actually a distended bladder. The patient is unable to empty his bladder secondary to an enlarged prostate that is obstructing the flow of urine. The incontinence is the result of the failure of the bladder to empty. Male patients who present with outlet obstruction secondary to BPH typically have urinary infrequency, slow weak stream, and small volume voids. The residual urine level is high. In women the cause of overflow incontinence is due to detrusor hypotonicity, with poor emptying, typically from diabetes mellitus, autonomic neuropathy, or anticholinergic medication. The patient with “overflow” usually only urinates the urine that “spills” out. They have a persistently elevated amount of urine in the bladder (post-void residual) and only urinate when their already elevated bladder capacity is surpassed.

Abnormal bladder sensation (choice A) is caused by a number of different possibilities. An interruption in the normal innervation of the bladder may prevent it from storing or emptying. Etiology includes multiple sclerosis, Parkinson’s disease, diabetes mellitus, or spinal injury. This patient does not provide a history of any of these nor does he have any physical findings that make any of these causes a high possibility.

Urge incontinence is caused by an involuntary rise in intravesical pressure secondary to detrusor contraction, which overcomes outlet resistance. These are called uninhibited bladder contractions (choice C). This can occur from local bladder causes such as infection, bladder stones, bladder tumors, or foreign bodies. It may also occur from a loss of cortical inhibition of the voiding reflex from stroke, dementia or Parkinson’s.

A vesicocutaneous fistula (choice D) occurs when there is a direct communication between the bladder and the skin. These patients constantly leak urine, frequently from a site other than the urethra. There is frequently a history of radiation, trauma, or manipulation.

Weakened pelvic floor muscles (choice E) is also called stress incontinence. It is characterized by the involuntary loss of urine associated with physical activities such as coughing, laughing, sneezing, or exercise. It usually occurs in female patients who have had multiple vaginal deliveries.

 

  A 71-year-old woman comes to the office for an initial visit. She recently moved into town to be close to her daughter after her husband died. Her past medical history is significant for depression and peripheral edema. She takes hydrochlorothiazide for the latter. Upon further questioning she admits to “bladder problems”, which she describes as occasional leakage of urine. Socially, she has 4 children, is widowed, and is a retired schoolteacher who enjoys the theater and concerts. She drinks a glass of wine with dinner and 2 cups of coffee per day. Physical examination reveals mild atrophic vaginitis but is otherwise unremarkable. When questioned about her urinary incontinence, the patient states that her problems began with the birth of her second child. Initially she noted only very occasional episodes of small amounts of urine leakage with a forceful cough or sneeze. Over the last several years these episodes have occurred more frequently with less and less strenuous activity. She denies any urinary urgency, decreased stream, dysuria, or hematuria. She is embarrassed to go out in public for fear of wetting herself. Urinalysis and postvoid residual urine measurement are within normal limits. The most appropriate initial management for this patient’s incontinence is to

  A. begin clean intermittent catheterization
  B. order lumbosacral MRI
  C. prescribe anticholinergics
  D. schedule bladder sling surgery
  E. teach pelvic floor muscle exercises
Explanation:

The correct answer is E. Prior to prescribing any form of treatment for this patient, it is important to identify and classify the type of incontinence that she suffers from. The patient is suffering from stress urinary incontinence (SUI). SUI is a loss of urine that occurs with physical activities that raise intraabdominal pressure (i.e., sneezing, coughing, laughing). It is usually not associated with abnormal detrusor contractions or urinary retention. It is caused by a weakening in the tissues that surround the urethra and bladder neck, which are then displaced downward during stress maneuvers. In women, estrogen loss leads to atrophy in the tissues that line and surround the urethra, bladder outlet, and vagina, which may cause weakening of the pelvic floor muscles and a decline in bladder outlet and urethral resistance. Topical estrogens may help improve atrophied vaginal tissue. Pelvic floor muscle exercises,( i.e., Kegel exercises), improve the urethral resistance by actively exercising the pubococcygeus muscle. This provides increased muscle support to the pelvic viscera and increased closing force on the urethra. There are a variety of methods to teach these exercises, but the idea is to attempt to have the patient stop their flow of urine midstream by contracting their pelvic floor muscles. Once it is confirmed that the patient is contracting the appropriate muscle groups, then the patient must contract and release these muscles for 10 seconds at a time throughout the day. This therapy is an excellent initial treatment modality, as it is easy, cheap, and when used properly, effective. Benefits are usually seen within 6 weeks. Other noninvasive techniques that may be utilized by patients with a variety of types of incontinence include biofeedback, bladder training, timed voiding, and behavioral therapy.

Clean intermittent catheterization (choice A) is utilized with patients who suffer from urinary retention and elevated post void residuals. In this process, a clean (not sterile) catheter is passed through the urethra into the bladder every 3 to 6 hours to empty the bladder. This patient has a normal post void residual, indicating that there is no evidence of urinary retention.

Less common causes of SUI include intrinsic urethral sphincter deficiency secondary to radiation, surgery, or trauma. Rarely, SUI may be caused by sacral cord defects. If this is suspected, then a MRI of the spine may be necessary. However, this is not a first-line intervention (choice B) and this patient is neurologically intact.

Urge incontinence is associated with a sudden strong urge to void and involuntary urination occurs prior to the patient reaching the bathroom. It is usually caused by involuntary detrusor contractions. If this occurs in a patient, who is neurologically intact, it is termed detrusor instability. When a causative neurological lesion is established, urge incontinence is termed detrusor hyperreflexia. Anticholinergics (choice C) are the first-line pharmacologic therapy for patients with urge incontinence. These agents work by increasing bladder capacity and causing remission of uninhibited bladder contractions. This patient is not suffering from urge incontinence, therefore anticholinergics are not indicated.

Bladder sling surgery (choice D) works by reestablishing the correct anatomical arrangement of the bladder and urethra. It is indicated in carefully selected patients and only after less, non-invasive treatment modalities have been attempted. Ultimately, this patient may be a candidate for surgical intervention. However, this is not first-line therapy.

 

You are called to the cardiac intensive care unit to evaluate a 73-year-old patient who underwent an emergent cardiac catheterization for an acute myocardial infarction yesterday. You are told that the patient has had a urine output of only 60 cc of urine over the last 12 hours. Prior to this, his urine output had been within normal limits. His temperature is 37 C (98.6 F), blood pressure is 130/70 mm Hg, pulse is 60/min, and respirations are 14/min. He has no complaints at this time and his physical examination is unremarkable. Reviewing the medical chart, you learn that his medical history is significant for benign prostatic hypertrophy and hypertension. You catheterize his bladder and get 20 cc of dark urine, which you send for urine analysis and culture. You deliver a 500 cc normal saline fluid bolus and start intravenous fluid at 150 cc/hour. After 4 hours his urine output does not improve. Laboratory studies show:

Sodium 144 mEq/dL
Potassium 5.3 mEq/dL
Chloride 98 mEq/dL
Bicarbonate 21 mEq/dL
BUN 28 mg/dL
Creatinine 2.2 mg/dL (*Admission Creatinine- 1.3 mg/dL)

 

Urinalysis
Color Muddy brown
Specific gravity 1.020
Osmolality 55 mOsmol/kg
Leukocyte esterase Negative
Nitrite Negative
Protein Trace
Blood Negative
Microscopic Many muddy brown granular casts
Urine eosinophils None

The most likely cause of this patient’s new condition is

  A. acute interstitial nephritis
  B. acute tubular necrosis
  C. postrenal azotemia
  D. prerenal azotemia
  E. rapidly progressive glomerulonephritis
Explanation:

The correct answer is B. This patient has acute tubular necrosis (ATN) and acute renal failure. Remember that when confronted with acute renal failure, the cause is divided into prerenal or “dry”, postrenal or “obstructed”, and intrarenal. The history and the urinalysis provide us with the correct diagnosis. Patients with ATN can have muddy, granular casts in their urine. They will also have an elevated BUN and creatinine, but in a ratio of less than 20:1 as is seen in patients with prerenal azotemia. The history of recent cardiac catheterization should also provide a clue since the patient received a large ionic dye load during the procedure to image his coronary vessels. Dye is a common precipitant of ATN and needs to be considered in anyone who has an elevated creatinine after a procedure that involves contrast dyes.

Acute interstitial nephritis (AIN) (choice A) is another cause of acute renal failure. It is associated with a transient maculopapular rash, fevers, eosinophiluria, hematuria, and white cell casts. Drugs account for most cases of AIN.

Postrenal azotemia (choice C) would also be reasonable in our patient since he has a history for BPH (which is the most common cause of obstruction). If a Foley catheter is placed and a large amount of urine is found in the bladder, obstruction is a likely cause of poor urine output. Since this patient had a virtually empty bladder, it would be unlikely that he has a postrenal cause of his renal failure.

Prerenal azotemia (choice D) should be your first thought when evaluating a patient with dressed urine output because it is the most common cause of acute renal failure. This patient was given a fluid challenge and still did not increase his urine output. Therefore, prerenal azotemia should be lower on the differential. Other clues that this patient is not prerenal is that his BUN to creatine ratio is less than 20:1 and his urinalysis revealed granular casts.

Rapidly progressive glomerulonephritis (choice E) is a rare cause of acute renal failure. It is associated with hypertension and edema. Urinalysis reveals dysmorphic red blood cells, red cell casts, and mild proteinuria.

 

A 27-year-old woman comes to the emergency department with “red urine” and a 7-hour history of severe right-sided flank and groin pain. She has no other past medical history and takes only oral contraceptive pills for medications. She had one episode of vomiting in that period of time. She appears otherwise healthy but in moderate discomfort. Her temperature is 37.0 C (98.6 F), blood pressure is 123/90 mm Hg, and pulse is 100/min. Her physical examinaiton is notable for mild suprapubic tenderness but no costovertebral angle tenderness. Her urine is dipstick positive for red blood cells. The most appropriate next step in the evaluation is a/an

  A. abdominal CT scan with contrast
  B. abdominal radiograph
  C. abdominal ultrasound
  D. laparoscopy
  E. stone-protocol abdominal CT
Explanation:

The correct answer is B. This patient clearly has nephrolithiasis. Typical presentations include hematuria, flank pain radiating to the groin, and a benign abdominal exam. Since most kidney stones are composed of calcium oxalate, they are radio-opaque and thus visible on plain radiographs. The remaining components of the diagnostic evaluation include analysis of the urine for crystals, blood, and pH.

An abdominal CT scan with contrast (choice A) is considered not sensitive enough to detect renal stones. However, in some centers, a stone-protocol abdominal CTs (choice E) are being increasingly utilized. The resolution of new CT scanners is such that when properly focused on the appropriate anatomy in high-resolution cuts (1.5mm vs. standard 5 or 10mm cuts), most stones, opaque or lucent can be visualized. However, plain radiographs still remain the first imaging procedure of choice.

An abdominal ultrasound (choice C) has no utility in the diagnosis of renal stones but is very useful in the diagnosis of gallstones.

A laparoscopy (choice D) provides visualization of the abdominal cavity and therefore is of no use in the diagnosis of intrarenal system stones.

 

  A 30-year-old woman comes to the clinic complaining of loss of control of urination and “dribbling” of urine. She has had recurrent urinary tract infections over the past 11 months, which were treated with antibiotics. She has been following safe sex practices and denies any history of sexually transmitted diseases, but complains of moderate pain during intercourse. Physical examination and pelvic examination are normal. The most appropriate next investigation in this patient is

  A. CT scan of the abdomen and pelvis
  B. laparoscopy
  C. pelvic sonography
  D. renal sonography
  E. voiding cystourethrogram
Explanation:

The correct answer is E. The classical triad of dribbling, dyspareunia, and dysuria suggests a urethral diverticulum. This woman may have a tender suburethral mass. Frequently there is have a history of frequent urinary track infections. Diagnosis is confirmed by urethroscopy or a voiding cystourethrogram.

A CT scan of the abdomen and pelvis (choice A) is unnecessary, as intraabdominal or pelvic tumors typically do not present with a classical triad of dyspareunia, dribbling, and dysuria.

Laparoscopy (choice B) is not indicated as no pelvic pathology like endometriosis is suspected.

Pelvic sonography (choice C) is not indicated as no associated pathology with the uterus and the ovary is suspected.

Renal sonography (choice D) may not show any evidence for a urethral diverticulum.

 

A 36-year-old woman comes to the emergency department with a 36-hour history of fever, chills, and right flank pain. She has had similar, but less severe episodes in the past which all improved with antibiotics and hydration. Her temperature is 39.1 C (102.4 F), blood pressure is 110/68 mm Hg, pulse is 115/min, and respirations are 24/min. Physical examination shows severe right sided costovertebral tenderness and mild right lower quadrant tenderness without rebound. She vomits twice during your exam. Urinalysis shows 18 red blood cells per high power field. A CT scan without contrast shows moderate right-sided hydronephrosis with perinephric stranding and a 7 mm stone in the mid right ureter. The appendix is not visualized. Laboratory studies show:

The most appropriate intervention at this time is

  A. extra-corporeal shock wave lithotripsy of ureteral stone
  B. intravenous hydration
  C. intravenous hydration and antibiotics
  D. intravenous hydration, antibiotics, and placement of right percutaneous nephrostomy tube
  E. observation alone
  F. repeat CT scan with oral contrast to evaluate for appendicitis
Explanation:

The correct answer is D. This patient is suffering from obstructive uropathy leading to pyonephrosis (distention of the renal pelvis with pus) and sepsis. Treatment of the infected, obstructed collecting system is priority to preventing further renal failure and sepsis. Relief of obstruction (decompression) can be performed with either a percutaneous nephrostomy tube or insertion of ureteral stent (the preferred modality is controversial with available data supporting either modality). Manipulation of an infected renal collecting system at the time of infection should be kept to a minimum, and treatment should be limited to procedures that allow for adequate drainage of the obstructed kidney.

ESWL (choice A) is contraindicated during infection. This therapy, which provides shock waves to renal/ureteral calcifications, would not provide relief of urinary obstruction.

Antibiotics and hydration (choices B and C) are important therapeutic modalities for this patient. However, they are not the definitive therapy and must be accompanied by decompression of the dilated kidney. Broad spectrum antibiotics should be implemented immediately and tailored according to cultures from the pus that is drained at time of nephrostomy tube placement. Without relief of obstruction, any antibiotic treatment will be incomplete.

Because this patient has had similar less severe episodes in the past, one might be tempted to observe her (choice E). However, while working in the emergency department it is important to realize that this patient exhibits none of the prerequisites that would make her eligible for observation (afebrile, no white count, no azotemia, able to tolerate oral intake and oral analgesics).

This patient is clearly very sick. While acute appendicitis (choice F) must always be included in the differential of a patient with right lower quadrant tenderness, fever, and elevated white blood cell count, this patient’s complete clinical picture is not consistent with appendicitis. Delaying definitive treatment for further radiologic studies would only prolong clinical improvement and would further endanger this patient’s health.

 

  A 7-year-old boy is brought to the office by his mother because of a 4-week history of wetting his bed at night. The mother tells you that he has been able to hold his urine at night since he was 4 years old, and that this “nightly bedwetting issue” began abruptly. He is able to stay dry during the day. She states that he is a very active child, and that the “bumps and bruises” on his arms, legs, and buttocks are his “battle wounds.” This is the first time you have ever seen this child. His temperature is 36.7 C (98.0 F). Physical examination is unremarkable. The most appropriate first step is to

  A. ask the mother and child separately what is going on in his environment
  B. call his teacher and ask about issues or problems at school
  C. obtain a urine culture
  D. order a voiding cystourethrogram
  E. prescribe imipramine
Explanation:

The correct answer is A. This patient has secondary enuresis, which is the involuntary discharge of urine after bladder control has been reached. One of the most important causes of secondary enuresis is a stressful environment that may include a new house or new school, the birth of a brother or sister, parental conflicts, or any other real or perceived life-altering events. It is usually necessary to talk to both the patient and parent individually to determine the cause. The patient may be more forthcoming when interviewed alone, and may verbalize complaints and concerns more easily.

Getting information from the school and his teacher may be important (choice B) but it is not the first step.

A urine culture (choice C) may be necessary. However, it is not the first step. It is best to first try to distinguish if the enuresis is due to an underlying organic, psychogenic, or functional cause. This may be done by questioning the patient and parent individually about a stressful environment. After a comprehensive questioning session with both individuals, a urinalysis and then possibly a urine culture should be obtained.

A voiding cystourethrogram (choice D) is usually performed to evaluate vesicoureteral reflux, which is a cause of urinary tract infections and pyelonephritis in children. It is not part of the initial evaluation for secondary enuresis.

It is inappropriate to prescribe imipramine (choice E) at this time. It is necessary to determine the cause of the enuresis before any treatment is initiated. It is important to first differentiate whether the patient’s condition is due to an underlying organic, psychogenic, or functional disorder. Behavior modification is usually the preferred treatment for enuresis, such as urinating before bed, avoiding liquids after supper, and positive reinforcement. Pharmacologic therapy with imipramine is not the first step because of drug tolerance, side effects, and other factors. Intranasal ddAVP, which is an antidiuretic hormone analog, is useful, but it does not address the underlying problem/issue.

 

A 52-year-old man is admitted to the hospital with pancreatitis. Prior to admission he states that he has had approximately 3 days of epigastric pain, nausea, and vomiting. He has not been able to keep down any oral intake during this time period. His past medical history is significant for cholelithiasis, but he has otherwise been very healthy. His temperature is 37.2 C (99 F), blood pressure is 102/58 mm Hg, pulse 102/min, and respirations 18/min. Laboratory studies show a leukocyte count 7,300mm3, platelets 335,000mm3, hematocrit 56%, BUN 85 mEq/L, creatinine 2.8 mEq/L, ALT 40 U/L, AST 35 U/L, alkaline phosphatase 85 U/L, bilirubin (total) 1.0 mg/dL, and amylase 750 U/L. The most likely treatment that will help his renal failure is

  A. a bolus of 1 L of normal saline and then reassessment
  B. methylprednisolone 125 mg intravenously followed by a steroid taper
  C. no treatment is necessary at this time
  D. place a Foley catheter and keep it to drainage
  E. a renal biopsy followed by treatment with cyclophosphamide
Explanation:

The correct answer is A. This patient is presenting with several days of nausea and vomiting with poor oral intake. He is tachycardic and his blood pressure is somewhat low. All of these point to the fact that he is volume depleted. His BUN and creatinine ratio is greater than 20:1, which suggests a prerenal cause of his renal failure. His high hematocrit also implies hemoconcentration. He will likely need a bolus of at least 1 liter of normal saline. However, it is prudent to give staged boluses of fluid and then reassess, to avoid volume overloading in the patient.

Methylprednisolone (choice B) and cyclophosphamide (choice E) are immunosuppressive drugs often used in renal failure caused by autoimmune diseases such as systemic lupus erythematosus. This clinical scenario is much more consistent with prerenal volume depletion induced renal dysfunction.

Clearly no treatment (choice C) is not an appropriate choice here.

Placing a Foley catheter (choice D) is an important thing to do initially in many people with acute renal failure, particularly older men with prostatic hypertrophy. This can relieve an obstruction causing postrenal failure. It can be argued that a Foley catheter should be placed in this patient. However, it is not likely to help treat the underlying problem.

 

A 6-year-old boy is brought to the office by his mother because of a “red rash” that she noticed today. She says that 3 days ago he had a cough, runny nose, and fever that responded to ibuprofen. In the office, his temperature is 37 C (98.6 F) and he has a normal physical examination with the exception of an erythematous, blanching macular rash on his legs. You diagnose him with a viral exanthem and advise the mother to encourage the child to drink liquids and to use ibuprofen as needed for fever. One week later, the mother brings the child back to the office and reports that the rash has “changed”, he has developed colicky abdominal pain several times per day, and he is complaining of left knee pain. In the office, his temperature is 37.2 C (99 F), blood pressure is 100/65 mm Hg, pulse is 100/min, and respiratory rate is 15/min. A physical examination reveals a well-appearing child with palpable purpura of both lower extremities, normal neck examination, clear lungs, and a soft, non-tender abdomen. His left knee is painful on flexion, but it is not erythematous or warm, and there does not seem to be an effusion. His gait is normal. The most appropriate study at this time is

  A. arthrocentesis
  B. colonoscopy
  C. cultures of blood, urine, and cerebrospinal fluid
  D. echocardiography
  E. urinalysis
Explanation:

The correct answer is E. This patient most likely has Henoch-Schonlein purpura (HSP), a small-vessel vasculitis seen most commonly in children between the ages of 2 and 8. The child’s preceding upper respiratory tract infection, low-grade fever, and arthralgias are all common elements of this disease. The typical rash of HSP is an evanescent, erythematous, macular rash on the lower extremities that progresses over the course of days to petechiae and palpable purpura. These change in color from red to purple to brown before eventually fading, normally over the course of weeks. HSP is an IgA-mediated autoimmune vasculitis, which can cause tissue damage as a result of immune complex formation. Deposition of these immune complexes in the kidneys can lead to nephritis, which is the leading cause of permanent sequelae from HSP. End-stage renal disease is an uncommon but possible outcome. It is important to perform frequent urinalyses for early detection of kidney involvement.

The joint manifestations of HSP are commonly arthralgias without arthritis, and can last weeks to months. Residual joint disease is rare, and arthrocentesis (choice A) is not necessary as part of the diagnostic evaluation.

Immune complex deposition in the bowel wall can lead to colicky abdominal pain in the setting of HSP and can result in serious, life-threatening sequelae (e.g., intussusception, perforation). This child’s abdomen is benign on examination, and his symptoms should be followed clinically. A colonoscopy (choice B) is not necessary at this time.

While petechiae and purpura can be manifestations of overwhelming infection, often due to N. meningitis, the time course of the disease, the well-appearance of the child, and his stable vital signs argue against an infectious cause of this child’s symptoms. Thus, cultures from multiple body sites (choice C) are not warranted at this time.

Echocardiography (choice D) is an integral part of the workup of Kawasaki’s disease because of the propensity of affected children to develop coronary artery aneurysms, but is not a concern in HSP.

 

  A 56-year-old diabetic patient returns to the office for counseling regarding his newly diagnosed nephrotic syndrome. On his last visit, he was diagnosed with nephrotic syndrome and was given a series of instructions and literature to read over in preparation for this visit. He is now ready to hear about his disease. It is appropriate to advise the patient that:

  A. He is at an increased risk for certain lymphomas and renal cell carcinoma
  B. His condition increases his risk for a coagulopathy with a risk of mild bleeds
  C. His condition will increase his susceptibility to bacterial infections
  D. This condition increases his risk for developing hypervolemia
  E. This condition is associated with a decreased cardiovascular risk from atherosclerosis
Explanation:

The correct answer is C. The nephrotic syndrome carries with it a host of attendant complications that are responsible for the majority of morbidity and mortality. For a variety of reasons, some not completely clear, these patients have an increased susceptibility to bacterial infections.

Increased risk of certain lymphomas and renal-cell carcinoma (choice A) has not been demonstrated in repeated epidemiological and longitudinal case studies of nephrotic syndrome patients.

Mild coagulopathy with risk of mild bleeds (choice B) is incorrect as nephrotic syndrome is actually a mildly hypercoagulable state marked by increased frequency of DVTs and pulmonary emboli.

Hypervolemia (choice D) is in fact the opposite of the normal problem which is hypovolemia. The nephrotic syndrome is a disease of decreased effective arterial blood volume that can lead to varying degrees of renal underperfusion. The massive third spacing of fluid is the primary reason why these patients are consistently intravascularly volume depleted.

Decreased cardiovascular risk from atherosclerosis (choice E) is incorrect. Nephrotic syndrome is associated with hyperlipidemia and thus an increased risk of CV complications.

 

A 39-year-old man with no significant past medical history comes to the office because of “red urine.” He tells you that he has been well over the past few years, but has occasionally noticed hematuria in the morning. His family history is unremarkable. He denies any tobacco or intravenous drug use. His blood pressure is 180/100 mm Hg and pulse is 70/min. Physical examination shows clear lungs, normal heart sounds without murmurs, and 3+ pitting edema of his lower extremities bilaterally. Laboratory studies show:

The most likely diagnosis is

  A. focal segmental glomerulosclerosis
  B. IgA nephropathy
  C. membranoproliferative glomerulonephritis
  D. rapidly progressive glomerulonephritis
  E. systemic lupus erythematosus
Explanation:

The correct answer is B. IgA nephropathy, also called Berger disease, is a major cause of recurrent glomerular hematuria. The hematuria typically lasts for a few days, and then subsides, only to recur every few months. The total picture is that of a nephrotic syndrome, with hyperlipidemia, proteinuria, and hypoalbuminemia. Most patients have an initial benign course, but up to 50% develop chronic renal failure by 20 years.

Focal segmental glomerulosclerosis (choice A) is a rare cause of idiopathic glomerular disease, usually characterized by hypertension, hematuria, renal insufficiency, and nephrosis. Compared to Berger disease, this entity in its primary form is quite rare.

Membranoproliferative glomerulonephritis (choice C) exhibits a variety of presentations ranging from asymptomatic proteinuria to full nephrotic syndrome. The diagnosis should be suspected when renal disease is accompanied by low serum complement levels. Hepatitis C virus accounts for most of the cases of MPGN.

Rapidly progressive glomerulonephritis (choice D) presents with rapidly declining renal function, an active urinary sediment including RBC casts, as well as oliguria or anuria.

Systemic lupus erythematosus (choice E) may involve the kidney and can have a variety of renal presentations; from benign progressive azotemia to rapidly progressive nephritis. The kidneys are involved in nearly all cases of SLE, but rarely without involvement of other organs.

 

 

  A 64-year-old man comes to the office complaining of left flank pain and gross hematuria. The pain is non-radiating, dull in nature, and not associated with any nausea or vomiting. The hematuria is painless, intermittent, and not associated with any fevers or chills. The patient has not had any medical care in over 20 years. His vital signs and physical examination are unremarkable. A complete blood count and biochemical profile are normal. A urinalysis shows full field red blood cells. A CT scan of abdomen and pelvis reveals a contrast enhancing 5cm mass in the lower pole of the left kidney. A chest x-ray shows no signs of metastasis. The most appropriate intervention at this time is to

  A. order a CT guided percutaneous needle biopsy
  B. order an MRI with gadolinium
  C. schedule chemotherapy
  D. schedule radiation therapy
  E. send him to a urologist for a radical nephrectomy
Explanation:

The correct answer is E. This patient has renal-cell carcinoma (RCC) of the left kidney. CT scan with and without contrast that reveals a contrast-enhancing mass is the primary radiological modality used to diagnose this disease. This patient presents with two of the three symptoms of the classic triad of RCC: costovertebral pain, hematuria, and palpable mass. Unfortunately this triad is only seen in 10% of patients diagnosed with RCC. Surgical removal remains the only potential curative therapy for RCC. While a variety of therapeutic modalities are available (i.e., partial nephrectomy), radical nephrectomy provides this patient with the best possible chance of cure. Prognosis for patients with T1 RCC (less than 7 cm in size) is >85% with appropriate treatment.

Percutaneous biopsy (choice A) has a limited role in the evaluation of RCC. Patients with radiologic findings consistent with RCC require definitive surgical management and should undergo biopsy only if there is evidence of metastatic disease. A histological diagnosis is required prior to consideration of a non-surgical therapy.

MRI with gadolinium (choice B) may be utilized for patients with contrast allergy, renal insufficiency (who cannot receive contrast), or for evaluation of vascular invasion of tumor. This patient does not meet any of these criteria. It is unlikely that a mass this size located in the lower pole would invade the renal vein.

Renal-cell carcinoma is very resistant to chemotherapy (choice C). It is not the appropriate treatment for a patient who is a candidate for curative therapy.

Radiation therapy (choice D) has a role in the palliative care of patients with metastatic RCC. There is no clinical, laboratory or radiographic evidence that this patient has metastatic disease.

 

 

A 39-year-old man comes to the office because of “achy low back pain” for the past few weeks. He states that it is a dull pain that came on gradually. He has no other symptoms. You have been treating him for hypertension with hydrochlorothiazide and enalapril for years. His father and younger sister also have hypertension. His temperature is 36.7 C (98.0 F), blood pressure is 135/90 mm Hg, and pulse is 65/min. Physical examination shows left-sided flank tenderness. Cardiac examination reveals a mid-systolic click. A urine dipstick test shows microscopic hematuria. The most appropriate next step is to

  A. arrange a renal biopsy
  B. order a CT scan of the kidneys
  C. order an intravenous pyelogram
  D. order a renal ultrasound
  E. reassure him and treat him with naproxen
  F. send him for genetic linkage analysis
Explanation:

The correct answer is D. This patient most likely has polycystic kidney disease and the most appropriate next step is to order a renal ultrasound, which will probably show multiple renal cysts. Many adult patients have hypertension, gross or microscopic hypertension, chronic flank pain, and chronic renal failure. It is usually an autosomal dominant disorder, which helps to explain the hypertension in his father and younger sister. This condition is associated with mitral valve prolapse, colonic diverticular disease, intracranial aneurysms, hepatic, splenic, and pancreatic cysts.

A renal biopsy (choice A) is not indicated in the evaluation of a patient with polycystic kidney disease.

A CT scan (choice B) is a sensitive diagnostic study used to evaluate patients with suspected polycystic kidney disease, however it is usually done after an ultrasound to detect smaller cysts.

An intravenous pyelogram (choice C) results in good visualization of the kidneys and urinary tract, however it requires the administration of contrast dye, and does not discriminate between cystic and solid masses. It is therefore not part of the initial evaluation of a patient with suspected polycystic kidney disease.

Reassurance and the treatment with naproxen (choice E) is appropriate for a patient that complains of low back pain without significant neurologic signs. It is not appropriate for this patient who actually has flank pain that he called low back pain and probably polycystic kidney disease.

Genetic linkage analysis (choice F) will establish the diagnosis of polycystic kidney disease, but is usually used in cases of suspected polycystic kidney disease when the radiographic imaging is negative.

 

A 7-year-old boy is brought to the emergency department by his mother because of “tea-colored urine” for the last several days. He has also had some nausea and vomiting, and his eyes appear swollen when he wakes up in the morning. The eye swelling tends to resolve over the course of the day. He is generally very healthy and there is no family history of any chronic diseases. His temperature is 36.7 C (98.0 F), blood pressure is 130/90 mm Hg, pulse is 96/min, and respiratory rate is 16/min. Physical examination is unremarkable. A urinalysis shows red cell casts. At this time the most appropriate study to confirm your diagnosis is

  A. antinuclear antibody
  B. antistreptolysin O antibody
  C. renal biopsy
  D. renal ultrasound
  E. urine culture
Explanation:

The correct answer is B. Poststreptococcal glomerulonephritis is the most common cause of acute glomerulonephritis in children. It usually follows a streptococcal pharyngitis by 1-2 weeks and a streptococcal skin infection by 2-3 weeks. It most commonly occurs in school-age children and has a male to female predominance of 2:1. It is most commonly characterized by hematuria (microscopic or gross) with red cell casts, proteinuria, hypertension (from fluid overload secondary to decreased glomerular filtration rate), and edema (from retention of salt and water). Laboratory values are usually significant for markedly decreased complement levels (C3 and C4), hypo- or hypernatremia, and a blood urea nitrogen elevated disproportionately to the creatinine. In order to diagnose poststreptococcal glomerulonephritis with certainty, there needs to be evidence of a preceding streptococcal infection such as an elevated ASO or streptozyme.

Systemic lupus erythematosus (SLE) nephritis is another cause of hematuria. If this child had persistently low complement levels, an ANA (choice A) would be indicated as one of the first screening tests for SLE. Keep in mind that only 25% of cases of SLE are diagnosed within the first two decades of life, the male-to-female ratio is 1:8, and renal disease is not present in all cases of SLE. Therefore, it is unlikely that this 7-year-old boy has new onset SLE, and other more common causes of glomerulonephritis must be ruled out first.

At this time, a renal biopsy (choice C) is not indicated. A renal biopsy is an invasive procedure with inherent risks. As this patient fits the perfect description for post streptococcal glomerulonephritis a renal biopsy would only be indicated if the complement level does not return to normal within 8 weeks, and the gross hematuria does not resolve over several weeks. If one was to do a renal biopsy on a patient with post-streptococcal glomerulonephritis, the findings would be: mesangial and capillary cell proliferation, inflammatory cell infiltration, and granular “humps” of IgG and C3 below the glomerular basement membrane.

A renal ultrasound (choice D) is useful in evaluating parenchymal disease, urinary tract abnormalities, or renal blood flow. In the case of post-streptococcal glomerulonephritis, the changes are microscopic and therefore would not be seen on ultrasound.

Urinary tract infections are a common cause of hematuria. Therefore, it is a good idea to do a urine culture (choice E) in all cases of hematuria. Unfortunately, the child in this case has red cell casts which indicate the kidneys are the source for the blood. So, although a urine culture should be sent on this child, it will probably not help with the diagnosis.

 

A 38-year-old man is admitted to the hospital for acute deterioration in renal function. He was seen in your office 2 days prior for some mild upper respiratory complaints, including a sore throat, cough, and fever. He was prescribed cephalexin and sent home. Today, his laboratory data returned and shows a blood urea nitrogen level of 67 mg/dL and a creatinine level of 2.1 mg/dL. You called him and told him to meet you at the hospital for further evaluation. On admission his BUN is now 109 mg/dL and his creatinine is 4.2 mg/dL. The appropriate tests are ordered and an electrocardiogram shows QRS complex widening and tall, peaked T waves. His temperature is 38.3 C (101.0 F). He has an erythematous oropharynx with some mild tonsillar exudate. His lungs are clear. It is observed that he has urinated only 5-10 cc in the past 2 hours since his hospitalization. A urinalysis shows red cell casts and dysmorphic red blood cells. The most appropriate next step is to

  A. administer high-dose methyl-prednisolone, intravenously
  B. administer low-dose methyl-prednisolone, intravenously
  C. administer penicillin, intravenously
  D. prescribe high-dose cyclophosphamide, orally
  E. prescribe penicillin, orally
Explanation:

The correct answer is A. The diagnosis and treatments for rapidly progressive glomerulonephritis, in this case, are woefully inadequate. In adults, where the disease is much less common, and the outcome much worse, early initiation of high-dose steroids has been associated with improved mortality and more rapid return of renal function. Even this therapy, however, has benefits for at most, 40% of patients.

For this reason, lower dose steroids (choice B) are not adequate therapy.

Intravenous penicillin (choice C) or oral penicillin (choice E) presume that treatment of the inciting infection will alter the course of the renal failure. This has not been the case in clinical practice and the severity of the precipitating illness and its duration are not correlated with the course of the glomerulonephritis.

Oral high-dose cyclophosphamide (choice D) is often reserved for more dramatic immune suppression in patients with more severe disease, but its efficacy and applicability to all RPGN patients is far from clear and therefore the risks associated with its use are not offset by any benefit.

 

  A 34-year-old woman comes to the office for a follow-up examination after passing a kidney stone in the hospital last week. You were away on vacation and so your partner was involved in her in-patient treatment. The patient tells you that your partner did not tell her anything about her condition and always seemed “as if he was late for his golf tee-off time.” Passing the stone was “more painful than the vaginal delivery of all 3 children combined” and so she wants to make sure that she never has one again. She has no chronic medical conditions, never had surgery, and takes no medications. Her father and brother both suffer from nephrolithiasis. You go over to the computer to check if the laboratory report on the composition of her kidney stone is complete. You see that the stone was composed of calcium and that she had a 24-hour urine collection done in the hospital that showed 295 mg of calcium and 15 mg of oxalate. Her serum calcium level is 8.5 mg/dL. The most appropriate course of action is to

  A. prescribe allopurinol, orally
  B. prescribe cholestyramine, orally, and a low-fat diet
  C. prescribe hydrochlorothiazide, orally
  D. recommend a high fluid intake, but no medications or supplements
  E. recommend megadoses of vitamin C
Explanation:

The correct answer is C. This patient most likely has idiopathic hypercalcuria, which is characterized by calcium kidney stones, normocalcemia, and unexplained hypercalcuria. It is believed to be hereditary and is more common in men. Hydrochlorothiazide has been shown to decrease urinary calcium levels and therefore decrease the rate of calcium stone formation.

Allopurinol (choice A) is a xanthine oxidase inhibitor that is used in patients with kidney stones composed of uric acid. It decreases serum and urinary uric acid levels.

Cholestyramine (choice B), an oxalate binding resin, combined with a low-fat diet, is used in patients with intestinal hyperoxaluria for the prevention of calcium stones. These patients have a 24-hour urinary oxalate level greater than 50 mg. It occurs in patients with prior bowel surgery and chronic diseases of the gastrointestinal tract that lead to malabsorption and bacterial overgrowth.

It is inappropriate to recommend a high fluid intake, with no medications or supplements (choice D) to a patient with idiopathic hypercalcuria and kidney stones. Thiazides have been shown to reduce the rate of kidney stone formation in a patient with her condition.

Megadoses of vitamin C (choice E) are controversial and have been associated with the formation of kidney stones. Megadoses of vitamin C are not generally recommended for the prevention of calcium stones in patients with idiopathic hypercalcuria.

 

A 39-year-old man comes to the clinic for follow up after a short hospital admission for an episode of renal colic. At the time of admission the patient had an intravenous pyelogram (IVP) performed that showed a mid-ureteral calculus on the left side. There was delayed uptake and excretion of contrast in the left kidney, consistent with obstruction. There was also a filling defect at the level of L5 consistent with the calcification seen on pre-contrast films. His pain persisted and he developed a low-grade temperature. He received a urologic consultation and evaluation. Now, the patient hands you the results from a urine culture taken in the hospital that had no growth. While in the hospital he underwent ureteroscopic stone extraction of the left mid-ureteral calculus. The stone was sent to the laboratory for chemical analysis. He has no significant medical history, he denies a prior history of renal stones, and is on no medications. The composition of this stone is most likely

  A. calcium oxalate
  B. cystine
  C. indinavir
  D. magnesium-ammonium-phosphate
  E. uric acid
Explanation:

The correct answer is A. This patient has a radio-opaque renal calculi. Calcium oxalate containing stones account for approximately 60% of all kidney stones. These stones are radio-opaque, (visible on plain x-ray films). Hypercalciuria is the direct precursor to most calcium stone formations. This increase of calcium levels in the urine can occur from increased intestinal absorption, decreased renal absorption, or increased bone resorption. Oxalate is either produced endogenously from the enzymatic cleavage of glyoxylate to oxalic acid and glycine or via intestinal absorption.

Cystine stones (choice B) form from cystinuria. Cystinuria is the result of an inherited autosomal recessive defect in the renal tubular reabsorption of 4 amino acids, which include cystine, ornithine, lysine, and arginine (COLA). They occur in acidic urine and are very rare. Patients frequently provide a family history of kidney stones.

Indinavir (choice C) is a protease inhibitor used in the treatment of HIV. Patients taking this medication are at an increased risk of forming renal stones. These stones are very difficult to treat, as they cannot be seen on plain x-ray or CT scans.

Struvite stones are composed of magnesium ammonium phosphate (choice D). These stones are infectious in etiology and are formed by urea-splitting organisms. The most common organisms are the Proteus species, Pseudomonas, and Klebsiella. They are sometimes called “triple phosphate” stones and occur in the setting of persistently high urinary pH (>7.2). These stones have a high association with neurogenic bladders and foreign bodies in the urinary tract. Because the stones contain numerous infective bacteria, which the antibiotics cannot penetrate, the stone must be removed for the infection to be cleared.

Uric acid stones (choice E) account for approximately 10% of renal stones. They are not visible on plain films. Uric acid is a product of purine metabolism and is excreted in the urine. These stones form in the face of low urine volume, low pH (acid urine), and high levels of urinary uric acid. The etiology of uric acid stones include hyperuricuria, gouty diathesis, myeloproliferative diseases, malignancy, and Crohns disease.